LSAT and Law School Admissions Forum

Get expert LSAT preparation and law school admissions advice from PowerScore Test Preparation.

 Administrator
PowerScore Staff
  • PowerScore Staff
  • Posts: 8916
  • Joined: Feb 02, 2011
|
#41174
Complete Question Explanation
(The complete setup for this game can be found here: lsat/viewtopic.php?t=15645)

The correct answer choice is (D)

If H is the sixth field harvested, then according to the second rule J must be harvested seventh or eighth, and then only M is available to fill the remainder of the seventh or eighth space (this is because all the other fields must be harvested before other fields, for example, K must be harvested before M, and L must be harvested before K):
  • pt48_d05_g2_q11.png
Consequently, answer choice (D) is correct.
You do not have the required permissions to view the files attached to this post.

Get the most out of your LSAT Prep Plus subscription.

Analyze and track your performance with our Testing and Analytics Package.